GI med Surg 2

अब Quizwiz के साथ अपने होमवर्क और परीक्षाओं को एस करें!

The nurse is reviewing the record of a client admitted to the nursing unit and notes that the client has a history of Laënnec's cirrhosis. Which question related to the client's history would be most important to ask? "Do you abuse alcohol?" "Do you have any known cardiac disease?" "Does your type of employment cause you to have exposure to chemicals?" "Have you ever been told that you have had obstruction to your biliary ducts?"

"Do you abuse alcohol?" Laënnec's cirrhosis results from long-term alcohol abuse; therefore, the question inquiring about alcohol abuse is most appropriate. Cardiac cirrhosis most commonly is caused by long-term right-sided heart failure. Exposure to hepatotoxins, chemicals, or infections or a metabolic disorder can cause postnecrotic cirrhosis. Biliary cirrhosis results from a decrease in bile flow and is most commonly caused by long-term obstruction of bile ducts.

The nurse is performing an assessment on a client with acute pancreatitis who was admitted to the hospital. Which assessment question would most specifically elicit information regarding the pain that is associated with acute pancreatitis? "Does the pain in your stomach radiate to your back?" "Does the pain in your lower abdomen radiate to your hip?" "Does the pain in your lower abdomen radiate to your groin?" "Does the pain in your stomach radiate to your lower middle abdomen?"

"Does the pain in your stomach radiate to your back?" The pain that is associated with acute pancreatitis is often severe, is located in the epigastric region, and radiates to the back. The remaining options are incorrect because they are not specific for the pain experienced by the client with pancreatitis.

The primary health care provider has determined that a client has contracted hepatitis A based on flu-like symptoms and jaundice. Which statement made by the client supports this medical diagnosis? "I have had unprotected sex with multiple partners." "I ate shellfish about 2 weeks ago at a local restaurant." "I was an intravenous drug abuser in the past and shared needles." "I had a blood transfusion 30 years ago after major abdominal surgery."

"I ate shellfish about 2 weeks ago at a local restaurant." Hepatitis A is transmitted by the fecal-oral route via contaminated water or food (improperly cooked shellfish), or infected food handlers. Hepatitis B, C, and D are transmitted most commonly via infected blood or body fluids, such as in the cases of intravenous drug abuse, history of blood transfusion, or unprotected sex with multiple partners.

A sexually active young adult client has developed viral hepatitis. Which client statement indicates the need for further teaching? "I should avoid drinking alcohol." "I can go back to work right away." "My partner should get the vaccine." "A condom should be used for sexual intercourse."

"I can go back to work right away." To prevent transmission of hepatitis, vaccination of the partner is advised. In addition, a condom is advised during sexual intercourse. Alcohol should be avoided because it is detoxified in the liver and may interfere with recovery. Rest is especially important until laboratory studies show that liver function has returned to normal. The client's activity is increased gradually, and the client should not return to work right away.

The nurse has given instructions to a client with hepatitis about postdischarge management during convalescence. The nurse determines that further teaching is needed if the client makes which statement? "I need to avoid alcohol and aspirin." "I should eat a high-carbohydrate, low-fat diet." "I can resume a full activity level within 1 week." "I need to take the prescribed amounts of vitamin K."

"I can resume a full activity level within 1 week." The client with hepatitis is easily fatigued and may require several weeks to resume a full activity level. It is important for the client to get adequate rest so that the liver can heal. The client should avoid hepatotoxic substances such as aspirin and alcohol. The client should take in a high-carbohydrate and low-fat diet. Vitamin K may be prescribed for prolonged clotting times.

A home care nurse visits a client who was recently diagnosed with cirrhosis. The nurse provides home care management instructions to the client. Which client statement indicates a need for further instruction? "I will obtain adequate rest." "I will take acetaminophen if I get a headache." "I should monitor my weight on a regular basis." "I need to include sufficient amounts of carbohydrates in my diet."

"I will take acetaminophen if I get a headache." Acetaminophen is avoided because it can cause fatal liver damage in the client with cirrhosis. Adequate rest and nutrition are important. The client's weight should be monitored on a regular basis. The diet should supply sufficient carbohydrates with a total daily calorie intake of 2000 to 3000.

A client with viral hepatitis is discussing with the nurse the need to avoid alcohol and states, "I'm not sure I can avoid alcohol." What is the most appropriate nursing response? "I don't believe that." "Everything will be all right." "I'm not sure that I understand. Would you please explain?" "I think you should talk more with the primary health care provider about this."

"I'm not sure that I understand. Would you please explain?" Explaining what is vague or clarifying the meaning of what has been said increases understanding for both the client and the nurse. Refusing to consider the client's ideas may cause the client to discontinue interaction with the nurse for fear of further rejection. False reassurance devalues the client's feelings. Placing the client's feelings on hold by referring him or her to the primary health care provider for further information is a block to communication.

The nurse is performing an assessment on a client with suspected acute pancreatitis. Which complaint made by the client supports the diagnosis? "I have epigastric pain radiating to my neck." "I have severe abdominal pain that is relieved after vomiting." "My temperature has been running between 96º F (35.5º C) and 97º F (36.1º C)." "I've been experiencing constant, severe abdominal pain that is unrelieved by vomiting."

"I've been experiencing constant, severe abdominal pain that is unrelieved by vomiting." Nausea and vomiting are common presenting manifestations of acute pancreatitis. A hallmark symptom is severe abdominal pain that is not relieved by vomiting. The vomitus characteristically consists of gastric and duodenal contents. Fever also is a common sign. Epigastric pain radiating to the neck area is not a characteristic symptom.

The nurse who is caring for a client with a diagnosis of cirrhosis is monitoring the client for signs of portal hypertension. Which finding should the nurse interpret as a sign or symptom of portal hypertension? Flat neck veins Abdominal distention Hemoglobin of 14.2 g/dL (142 mmol/L) Platelet count of 600,000 mm3 (600 × 109/L)

Abdominal distention With portal hypertension, proteins shift from the blood vessels via the larger pores of the sinusoids (capillaries) into the lymph space. When the lymphatic system is unable to carry off the excess proteins and water, they leak through the liver capsule into the peritoneal cavity. This is called ascites, and abdominal distention would be the consequence. Increased portal pressure can lead to findings associated with right-sided heart failure, such as distended jugular veins.

The nurse is developing a teaching plan for a client with viral hepatitis. The nurse should plan to include which information in the teaching session? The diet should be low in calories. Meals should be large to conserve energy. Activity should be limited to prevent fatigue. Alcohol intake should be limited to 2 ounces per day.

Activity should be limited to prevent fatigue. Rest is necessary for the client with hepatitis, and the client with viral hepatitis should limit activity to avoid fatigue. The diet should be optimal in calories, proteins, and carbohydrates. The client should take in several small meals per day. Alcohol is strictly forbidden.

The nurse has taught the client with chronic pancreatitis about risk factor modification to reduce the incidence of recurrences. The nurse determines that teaching was effective if the client states that it will be necessary to control which factor? Alcohol intake Ulcer Crohn's disease Diabetes mellitus

Alcohol intake Chronic pancreatitis is aggravated by continued alcohol intake. Each of the other options is not specifically associated with pancreatitis.

The nurse is reviewing the record of a client with a diagnosis of cirrhosis and notes that there is documentation of the presence of asterixis. How should the nurse assess for its presence? Dorsiflex the client's foot. Measure the abdominal girth. Ask the client to extend the arms. Instruct the client to lean forward.

Ask the client to extend the arms. Asterixis is irregular flapping movements of the fingers and wrists when the hands and arms are outstretched, with the palms down, wrists bent up, and fingers spread. Asterixis is the most common and reliable sign that hepatic encephalopathy is developing. Options 1, 2, and 4 are incorrect.

A client with viral hepatitis states, "I am so yellow." What is the most appropriate nursing action? Assist the client in expressing feelings. Restrict visitors until the jaundice subsides. Perform most of the activities of daily living for the client. Provide information to the client only when he or she requests it.

Assist the client in expressing feelings. The client should be supported to explore feelings about the disease process and altered appearance so that appropriate interventions can be planned. Restricting visitors would reinforce the client's negative self-esteem. To assist the client in adapting to changes in appearance, it is important for the nurse to encourage participation in self-care to foster independence and self-esteem. The client should be encouraged to ask questions to clarify misconceptions, to learn ways to prevent the spread of hepatitis, to reduce fear, and to make appropriate decisions.

The nurse is giving instructions to a client with cholecystitis about food to exclude from the diet. Which food item identified by the client indicates that the educational session was successful? Fresh fruit Brown gravy Fresh vegetables Poultry without skin

Brown gravy The client with cholecystitis should decrease overall intake of dietary fat. Foods that should be avoided include sausage, gravies, fatty meats, fried foods, products made with cream, and desserts. Appropriate food choices include fruits and vegetables, fish, and poultry without skin.

The nurse is caring for a client with a resolved intestinal obstruction who has a nasogastric tube in place. The primary health care provider has now prescribed that the nasogastric tube be removed. What is the priority nursing assessment prior to removing the tube? Checking for normal serum electrolyte levels Checking for normal pH of the gastric aspirate Checking for proper nasogastric tube placement Checking for the presence of bowel sounds in all 4 quadrants

Checking for the presence of bowel sounds in all 4 quadrants Distention, vomiting, and abdominal pain are a few of the symptoms associated with intestinal obstruction. Nasogastric tubes may be used to remove gas and fluid from the stomach, relieving distention and vomiting. Bowel sounds return to normal as the obstruction is resolved and normal bowel function is restored. Discontinuing the nasogastric tube before normal bowel function may result in a return of the symptoms, necessitating reinsertion of the nasogastric tube. Serum electrolyte levels, pH of the gastric aspirate, and tube placement are important assessments for the client with a nasogastric tube in place but would not assist in determining the readiness for removing the nasogastric tube.

The nurse is providing dietary instructions to a client hospitalized for pancreatitis. Which food should the nurse instruct the client to avoid? Chili Bagel Lentil soup Watermelon

Chili The client with pancreatitis needs to avoid alcohol, coffee and tea, spicy foods, and heavy meals, which stimulate pancreatic secretions, producing attacks of pancreatitis. The client is instructed in the benefit of eating small, frequent meals that are high in protein, low in fat, and moderate to high in carbohydrates.

The nurse should incorporate which in the dietary plan to ensure optimal nutrition for the client during the acute phase of hepatitis? Select all that apply. Select foods high in protein content. Consume multiple small meals throughout the day. Select foods low in carbohydrates to prevent nausea. Allow the client to select foods that are most appealing. Eliminate fatty foods from the meal trays until nausea subsides. Eat a nutritious dinner because it is typically the best tolerated meal of the day.

Consume multiple small meals throughout the day. Allow the client to select foods that are most appealing. Eliminate fatty foods from the meal trays until nausea subsides. Because the client with hepatitis experiences general malaise, small, more frequent meals are better tolerated than large meals, with breakfast being the best tolerated meal of the day. Self-selection of foods may enhance appetite over randomly selected foods. Fatty foods can exacerbate nausea and need to be avoided during the acute phase. The diseased liver may be unable to metabolize large amounts of protein at this time. The client should receive a diet high in carbohydrates to assist with meeting increased caloric needs. Anorexia typically increases as the day goes on.

Michael, 42 y.o. The man is admitted to the med-surg floor with a diagnosis of acute pancreatitis. His BP is 136/76, pulse 96, Resps 22, and temp 101. His past history includes hyperlipidemia and alcohol abuse. The doctor prescribes an NG tube. Before inserting the tube, you explain the purpose to the patient. Which of the following is the most accurate explanation? A. "It empties the stomach of fluids and gas." B. "It prevents spasms at the sphincter of Oddi." C. "It prevents air from forming in the small intestine and large intestine." D. "It removes bile from the gallbladder."

Correct Answer: A. "It empties the stomach of fluids and gas." An NG tube is inserted into the patient's stomach to drain fluid and gas. Nasogastric tubes are part of the standard of care in treating intestinal obstruction and can also be used to provide nutritional support. They are most common in surgical patients but are useful in any patient population where gastric decompression or nutritional support is necessary. Option B: An NGT does not prevent spasms at the sphincter of Oddi. The most common indication for placement of a nasogastric tube is to decompress the stomach in the setting of distal obstruction. Small bowel obstruction from adhesions or hernias, ileus, obstructing neoplasms, volvulus, intussusception, and many other causes may block the normal passage of bodily fluids such as salivary, gastric, hepatobiliary, and enteric secretions. Option C: Similarly, intractable nausea or emesis, whether caused by medications, intoxication, or other reasons, can be an indication for the placement of a nasogastric tube in order to prevent aspiration. Prophylactic placement of the NG tube in patients with abdominal surgery is not recommended. Patients who develop postoperative ileus tend to recover faster without the placement of an NG tube. Option D: Placement of an NGT does not remove bile from the gallbladder. NG tubes have been used for various reasons in patients with GI bleeding. In the past, NG lavage was thought to help control GI bleeding. However, recent studies have shown that this is not helpful. Another indication for placement of a nasogastric tube is in the setting of massive hematochezia.

Which of the following factors can cause hepatitis A? A. Contact with infected blood. B. Blood transfusions with infected blood. C. Eating contaminated shellfish. D. Sexual contact with an infected person.

Correct Answer: C. Eating contaminated shellfish. Hepatitis A can be caused by consuming contaminated water, milk, or food — especially shellfish from contaminated water. The most common mode of transmission of hepatitis A is via the fecal-oral route from contact with food, water, or objects contaminated by fecal matter from an infected individual. It is more commonly encountered in developing countries where due to poverty and lack of sanitation, there is a higher chance of fecal-oral spread. Option A: Hepatitis B is caused by blood and sexual contact with an infected person. Transfusion of blood and blood products, injection drug use with shared needles, needlesticks, or wounds caused by other instruments in healthcare workers and hemodialysis are all examples of parenteral and percutaneous exposures, but parenteral mode remains the dominant mode of transmission both globally and in the United States. Option B: Hepatitis C is usually caused by contact with infected blood, including receiving blood transfusions. Transmission can be parenteral, perinatal, and sexual, with the most common mode being the sharing of contaminated needles among IV drug users. Also, other high-risk groups include people who require frequent blood transfusions and organ transplantation of organs from infected donors. Option D: Hepatitis B and C can be caused by sexual contact with an infected person. Intravenous drug users, men who have sex with men, healthcare workers with exposure to infected body fluids, patients who require frequent and multiple blood transfusions, people who have multiple sexual partners, prisoners, partners of hepatitis B virus carriers, and persons born in endemic areas are all at high risk for hepatitis B virus infection.

A female client who has just been diagnosed with hepatitis A asks, "How could I have gotten this disease?" What is the nurse's best response? A. "You may have eaten contaminated restaurant food." B. "You could have gotten it by using I.V. drugs." C. "You must have received an infected blood transfusion." D. "You probably got it by engaging in unprotected sex."

Correct Answer: A. "You may have eaten contaminated restaurant food." Hepatitis A virus typically is transmitted by the oral-fecal route — commonly by consuming food contaminated by infected food handlers. The hepatitis A virus (HAV) is a common infectious etiology of acute hepatitis worldwide. HAV is most commonly transmitted through the oral-fecal route via exposure to contaminated food, water, or close physical contact with an infectious person. The virus isn't transmitted by the I.V. route, blood transfusions, or unprotected sex. Option B: Hepatitis B can be transmitted by I.V. drug use. In the United States, estimates are about 2.2 million people have chronic hepatitis B virus infection. It is transmitted parenterally and sexually when individuals come in contact with mucous membranes or body fluids of infected individuals. Option C: Hepatitis B can be transmitted by blood transfusion. Transfusion of blood and blood products, injection drug use with shared needles, needlesticks, or wounds caused by other instruments in healthcare workers and hemodialysis are all examples of parenteral and percutaneous exposures, but parenteral mode remains the dominant mode of transmission both globally and in the United States. Option D: Hepatitis C can be transmitted by unprotected sex. Transmission can be parenteral, perinatal, and sexual, with the most common mode being the sharing of contaminated needles among IV drug users. Also, other high-risk groups include people who require frequent blood transfusions and organ transplantation of organs from infected donors.

Stephen is a 62 y.o. patient that has had a liver biopsy. Which of the following groups of signs alert you to a possible pneumothorax? A. Dyspnea and reduced or absent breath sound over the right lung. B. Tachycardia, hypotension, and cool, clammy skin. C. Fever, rebound tenderness, and abdominal rigidity. D. Redness, warmth, and drainage at the biopsy site.

Correct Answer: A. Dyspnea and reduced or absent breath sounds over the right lung Signs and symptoms of pneumothorax include dyspnea and decreased or absent breath sounds over the affected lung (right lung). A pneumothorax is defined as a collection of air outside the lung but within the pleural cavity. It occurs when air accumulates between the parietal and visceral pleura inside the chest. The air accumulation can apply pressure on the lung and make it collapse. Option B: The risk of fatal hemorrhage in patients without malignant disease is 0.04%, and the risk of nonfatal hemorrhage is 0.16%. In those with malignancy, the risk of nonfatal hemorrhage is 0.4% and 0.57% for nonfatal hemorrhage. Option C: This can occur with the inadvertent puncture of the gallbladder or in patients with obstructive jaundice and dilated bile ducts. It usually presents with abdominal pain, fever, leukocytosis. It can also be painless in some patients. Biliary scintigraphy demonstrates the leak. Treatment is usually with fluids and antibiotics. Very rarely, endoscopic procedures like ERCP or surgery may be required. Option D: This is usually clinically insignificant except in patients with obstructive jaundice like primary sclerosing cholangitis or in the post-transplant setting. Currently, there is no recommendation for treating with prophylactic antibiotics, and treatment can be offered on a case by case basis.

You're caring for Jane, a 57 y.o. patient with liver cirrhosis who developed ascites and requires paracentesis. Before her paracentesis, you instruct her to: A. Empty her bladder. B. Lie supine in bed. C. Remain NPO for 4 hours. D. Clean her bowels with an enema.

Correct Answer: A. Empty her bladder. A full bladder can interfere with paracentesis and be punctured inadvertently. The preferred site for the procedure is in either the lower quadrant of the abdomen lateral to the rectus sheath. Placing the patient in the lateral decubitus position can aid in identifying fluid pockets in patients with lower fluid volumes. Ask the patient to empty his or her bladder before starting the procedure. Option B: Placing the patient in the lateral decubitus position can aid in identifying fluid pockets in patients with lower fluid volumes. Paracentesis is done in a lateral decubitus or supine position. The ascites fluid level is percussed, and a needle is inserted either in the midline or lateral lower quadrant (lateral to rectus abdominis muscle, 2 cm to 4 cm superomedial to anterior superior iliac spine). This positioning avoids puncture of the inferior epigastric arteries. Option C: NPO is not necessary for the procedure. There are few absolute contraindications for paracentesis. Coagulopathy and thrombocytopenia (both very common in cirrhotic patients) are themselves not absolute contraindications as the incidence of bleeding complications from the procedure has been shown to be very low. Option D: An enema is not necessary for the procedure. A bedside ultrasound should be used to identify an appropriate location for the procedure. Ultrasound can confirm the presence of fluid and identify an area with a sufficient amount of fluid for aspiration, thereby decreasing the incidence of both unsuccessful aspiration and complications.

Dr. Smith has determined that the client with hepatitis has contracted the infection from contaminated food. The nurse understands that this client is most likely experiencing what type of hepatitis? A. Hepatitis A B. Hepatitis B C. Hepatitis C D. Hepatitis D

Correct Answer: A. Hepatitis A Hepatitis A is transmitted by the fecal-oral route via contaminated food or infected food handlers. The most common mode of transmission of hepatitis A is via the fecal-oral route from contact with food, water, or objects contaminated by fecal matter from an infected individual. It is more commonly encountered in developing countries where due to poverty and lack of sanitation, there is a higher chance of fecal-oral spread. Option B: Hepatitis B is transmitted parenterally and sexually when individuals come in contact with mucous membranes or body fluids of infected individuals. Transfusion of blood and blood products, injection drug use with shared needles, needlesticks, or wounds caused by other instruments in healthcare workers and hemodialysis are all examples of parenteral and percutaneous exposures, but parenteral mode remains the dominant mode of transmission both globally and in the United States. Option C: Transmission of Hepatitis C can be parenteral, perinatal, and sexual, with the most common mode being the sharing of contaminated needles among IV drug users. Also, other high-risk groups include people who require frequent blood transfusions and organ transplantation of organs from infected donors. Option D: Hepatitis D is an RNA virus and a single species in the Deltavirus genus. It contains the hepatitis D antigen and RNA strand and uses HBsAg as its envelope protein; therefore, those who get hepatitis D virus infection have coinfection with the hepatitis B virus as well. Hepatitis D virus has similar modes of transmission as the hepatitis B virus, but perinatal transmission is uncommon.

A patient with chronic alcohol abuse is admitted with liver failure. You closely monitor the patient's blood pressure because of which change that is associated with liver failure? A. Hypoalbuminemia B. Increased capillary permeability C. Abnormal peripheral vasodilation D. Excess renin release from the kidneys

Correct Answer: A. Hypoalbuminemia Blood pressure decreases as the body is unable to maintain normal oncotic pressure with liver failure, so patients with liver failure require close blood pressure monitoring. Increased capillary permeability, abnormal peripheral vasodilation, and excess renin released from the kidneys aren't direct ramifications of liver failure. Option B: Once ascites is present, most therapeutic modalities are directed on maintaining negative sodium balance, including salt restriction, bed rest, and diuretics. Paracentesis and albumin infusion is applied to tense ascites. Transjugular intrahepatic portosystemic shunt is considered for refractory ascites. With worsening of liver disease, fluid retention is associated with other complications; such as spontaneous bacterial peritonitis. Option C: Hepatorenal syndrome is a state of functional renal failure in the setting of low cardiac output and impaired renal perfusion. Its management is based on drugs that restore normal renal blood flow through peripheral arterial and splanchnic vasoconstriction, renal vasodilation, and/or plasma volume expansion. However, the definitive treatment is liver transplantation. Option D: The most acceptable theory for ascites formation is peripheral arterial vasodilation leading to underfilling of circulatory volume. This triggers the baroreceptor-mediated activation of the renin-angiotensin-aldosterone system, sympathetic nervous system, and nonosmotic release of vasopressin to restore circulatory integrity. The result is an avid sodium and water retention, identified as a pre ascitic state. This condition will evolve in overt fluid retention and ascites, as the liver disease progresses.

A clinical manifestation of acute pancreatitis is epigastric pain. Your nursing intervention to facilitate relief of pain would place the patient in a: A. Knee-chest position B. Semi-Fowler's position C. Recumbent position D. Low-Fowler's position

Correct Answer: A. Knee-chest position Flexion of the trunk lessens the pain and decreases restlessness. Promote position of comfort on one side with knees flexed, sitting up, and leaning forward. Reduces abdominal pressure and tension, providing some measure of comfort and pain relief. Other positions do not decrease the pain. Option B: The Semi-Fowler's position is a position in which a patient, typically in a hospital or nursing home is positioned on their back with the head and trunk raised to between 15 and 45 degrees, although 30 degrees is the most frequently used bed angle. Option C: The word "lateral" means "to the side," while "recumbent" means "lying down." In the right or left lateral recumbent position, the individual is lying on their right or left side. This position makes it easier to access a patient's side. Option D: Supine position often increases pain. The Semi-Fowler's position is often used for purposes similar to those of the regular Fowler's position, including feeding and lung expansion, cardiac or respiratory conditions, and for patients with a nasogastric tube.

A patient with severe cirrhosis of the liver develops hepatorenal syndrome. Which of the following nursing assessment data would support this? A. Oliguria and azotemia B. Metabolic alkalosis C. Decreased urinary concentration D. Weight gain of less than 1 lb per week

Correct Answer: A. Oliguria and azotemia Hepatorenal syndrome is a functional disorder resulting from a redistribution of renal blood flow. Oliguria and azotemia occur abruptly as a result of this complication. Confusion due to hepatic encephalopathy is likely the last and most severe stage of liver disease as a result of the liver failing to break down toxic metabolites. Most importantly these patients notice they urinate less frequently in smaller and smaller volumes as they become oliguric. Option B: Excess organic acids are not being excreted by the damaged kidneys, resulting in an elevated concentration of hydrogen ions; decreased pH occurs, causing metabolic acidosis. Cirrhosis and portal hypertension can trigger the neurohormonal cascade which leads to the development of HRS. This, in turn, causes the production and release of vasodilators and cytokines like nitric oxide and prostaglandins which cause splanchnic and systemic vasodilation. Option C: Concentration of the urine is increased with decreased renal function. The systemic drop in circulating pressure triggers the carotid and aortic arch baroreceptors to activate three separate compensatory mechanisms. These include the renin-angiotensin-aldosterone system, vasopressin release, and activation of the sympathetic nervous system (SNS). Option D: With renal insufficiency, significant weight gain is expected due to fluid retention. The progression of cirrhosis causes a fall in cardiac output and a fall in systemic vascular resistance in a cycle that induces further renal vasoconstriction. This leads to further renal hypoperfusion, worsened by renal vasoconstriction with the endpoint of renal failure.

A male client with cholelithiasis has a gallstone lodged in the common bile duct. When assessing this client, the nurse expects to note: A. Yellow sclera B. Light amber urine C. Circumoral pallor D. Black, tarry stools

Correct Answer: A. Yellow sclera Yellow sclera may be the first sign of jaundice, which occurs when the common bile duct is obstructed. Jaundice can be a sign of a common bile duct obstruction from an entrapped gallstone. In the presence of jaundice and abdominal pain, often, a procedure is an indication to go and retrieve the stone to prevent further sequelae. Option B: Urine normally is light amber. Usually, patients with symptoms from gallstones present with right upper abdominal pain after eating greasy or spicy foods. There is often nausea and vomiting. Pain can also be present in the epigastric area that radiates to the right scapula or mid-back. Option C: Circumoral pallor doesn't occur in common bile duct obstruction; it is a sign of hypoxia, respectively. The classic physical exam finding is a positive Murphy's sign, where the pain is elicited on deep palpation to the right upper quadrant underneath the rib cage upon deep inspiration. Option D: Black, tarry stools don't occur in common bile duct obstruction; they are signs of GI bleeding. Progression of this condition is indicated by neurologic changes and hypotension (Reynold's pentad). Other sequelae are acute pancreatitis with symptoms of midepigastric pain and intractable vomiting.

Develop a teaching care plan for Angie who is about to undergo a liver biopsy. Which of the following points do you include? A. "You'll need to lie on your stomach during the test." B. "You'll need to lie on your right side after the test." C. "During the biopsy, you'll be asked to exhale deeply and hold it." D. "The biopsy is performed under general anesthesia."

Correct Answer: B. "You'll need to lie on your right side after the test." After a liver biopsy, the patient is placed on the right side to compress the liver and to reduce the risk of bleeding or bile leakage. The risk of fatal hemorrhage in patients without malignant disease is 0.04%, and the risk of nonfatal hemorrhage is 0.16%. In those with malignancy, the risk of nonfatal hemorrhage is 0.4% and 0.57% for nonfatal hemorrhage. Option A: The patient is usually kept in the right decubitus position. The duration of observation varies across centers ranging from 1 hour to 6 hours. The American Association for the Study of Liver Diseases guidelines recommends observation for 2 to 4 hours. The vital signs are monitored every 15 minutes for the first hour, every 30 minutes for the next hour, and hourly till discharge. Option C: The patients are made to lie in a comfortable supine position. The right hand is placed under the head in a neutral position. By percussion, the area of maximum dullness is identified over the right hemithorax. This is typically between the 6 and 9 intercostal spaces between the anterior and the midclavicular line. Option D: The skin is prepped and draped in a sterile fashion. The overlying skin is anesthetized using 1% lidocaine. The peritoneum is also anesthetized by inserting the needle along the upper border of the rib avoiding vascular structures.

Which of the following will the nurse include in the care plan for a client hospitalized with viral hepatitis? A. Increase fluid intake to 3000 ml per day B. Adequate bed rest C. Bland diet D. Administer antibiotics as ordered

Correct Answer: B. Adequate bed rest. Treatment of hepatitis consists of bed rest during the acute phase to reduce metabolic demands on the liver, thus increasing blood supply and cell regeneration. Institute bed red or chair rest during the toxic state. Provide a quiet environment; limit visitors as needed. Promotes rest and relaxation. Available energy is used for healing. Activity and an upright position are believed to decrease hepatic blood flow, which prevents optimal circulation to the liver cells. Option A: Monitor I&O, compare with periodic weight. Note enteric losses: vomiting and diarrhea. Diarrhea may be due to transient flu-like response to viral infection or may represent a more serious problem of obstructed portal blood flow with vascular congestion in the GI tract, or it may be the intended result of medication use (neomycin, lactulose) to decrease serum ammonia levels in the presence of hepatic encephalopathy. Option C: Encourage intake of fruit juices, carbonated beverages, and hard candy throughout the day. Monitor dietary intake and caloric count. Suggest several small feedings and offer the "largest" meal at breakfast. Large meals are difficult to manage when a patient is anorexic. Anorexia may also worsen during the day, making intake of food difficult later in the day. Option D: Establish isolation techniques for enteric and respiratory infections according to infection guidelines and policy. Encourage or model effective handwashing. Prevents transmission of viral disease to others. Thorough hand washing is effective in preventing virus transmission.

A male client has just been diagnosed with hepatitis A. On assessment, the nurse expects to note: A. Severe abdominal pain radiating to the shoulder. B. Anorexia, nausea, and vomiting. C. Eructation and constipation. D. Abdominal ascites.

Correct Answer: B. Anorexia, nausea, and vomiting. Hallmark signs and symptoms of hepatitis A include anorexia, nausea, vomiting, fatigue, and weakness. Acute hepatitis usually presents as a self-limited illness; development of fulminant hepatitis is rare. Typical symptoms of acute infection include nausea, vomiting, abdominal pain, fatigue, malaise, poor appetite, and fever; management is with supportive care. Option A: Abdominal pain may occur but doesn't radiate to the shoulder. Extrahepatic manifestations rarely occur but may include pancreatitis, rash, acute kidney injury with interstitial nephritis or glomerulonephritis, pneumonitis, pericarditis, hemolysis, and acute cholecystitis. Option C: Eructation and constipation are common in gallbladder disease, not hepatitis A. Patients may develop dark urine and pale stools within a week, followed by jaundice, icteric (yellow-tinted) sclera, and pruritus. Patients usually have elevated levels of serum alanine aminotransferase, aspartate aminotransferase, bilirubin, alkaline phosphatase, and lambda-glutamyl transpeptidase. Option D: Abdominal ascites is a sign of advanced hepatic disease, not an early sign of hepatitis A. Ascites is the pathologic accumulation of fluid within the peritoneal cavity. It is the most common complication of cirrhosis and occurs in about 50% of patients with decompensated cirrhosis in 10 years. The development of ascites denotes the transition from compensated to decompensated cirrhosis.

For a client with hepatic cirrhosis who has altered clotting mechanisms, which intervention would be most important? A. Allowing complete independence of mobility B. Applying pressure to injection sites C. Administering antibiotics as prescribed D. Increasing nutritional intake

Correct Answer: B. Applying pressure to injection sites. The client with cirrhosis who has altered clotting is at high risk for hemorrhage. Prolonged application of pressure to injection or bleeding sites is important. Instruct patient/SO of signs and symptoms that warrant notification of health care provider: increased abdominal girth; rapid weight loss/gain; increased peripheral edema; increased dyspnea, fever; blood in stool or urine; excess bleeding of any kind; jaundice. Option A: Complete independence may increase the client's potential for injury, because an unsupervised client may injure himself and bleed excessively. Instruct SO to notify health care providers of any confusion, untidiness, night wandering, tremors, or personality change. Changes (reflecting deterioration) may be more apparent to SO, although insidious changes may be noted by others with less frequent contact with the patient. Option C: Antibiotics are important to promote liver regeneration. However, they are not most important for a client at high risk for hemorrhage. Some drugs are hepatotoxic (especially narcotics, sedatives, and hypnotics). In addition, the damaged liver has a decreased ability to metabolize all drugs, potentiating cumulative effect and/or aggravation of bleeding tendencies. Option D: Encourage the patient to eat; explain reasons for the types of diet. Feed the patient if tiring easily, or have SO assisted the patient. Include the patient in meal planning to consider his/her preferences in food choices. Improved nutrition and diet are vital to recovery. The patient may eat better if the family is involved and preferred foods are included as much as possible.

When teaching a client about pancreatic function, the nurse understands that pancreatic lipase performs which function? A. Transports fatty acids into the brush border B. Breaks down fat into fatty acids and glycerol C. Triggers cholecystokinin to contract the gallbladder D. Breaks down protein into dipeptides and amino acids

Correct Answer: B. Breaks down fat into fatty acids and glycerol. Lipase hydrolyses or breaks down fat into fatty acids and glycerol. Lipase is an enzyme that breaks down triglycerides into free fatty acids and glycerol. Lipases are present in pancreatic secretions and are responsible for fat digestion. There are many different types of lipases; for example, hepatic lipases are in the liver, hormone-sensitive lipases are in adipocytes, lipoprotein lipase is in the vascular endothelial surface, and pancreatic lipase in the small intestine. Option A: Lipase is not involved with the transport of fatty acids into the brush border. Lipases are enzymes that play a crucial role in lipid transport. Hepatic lipase plays a crucial role in the formation and delivery of low-density lipoprotein(LDL). LDL is formed by the modification of intermediate density lipoprotein in the peripheral tissue and liver by hepatic lipase. These LDL particles are taken up, or endocytosed, via receptor-mediated endocytosis by target cell tissue. LDL serves to ultimately transport cholesterol from the liver to peripheral tissue. Option C: Fat itself triggers cholecystokinin release. Hormone-sensitive lipase is found within fat tissue and is responsible for degrading the triglycerides that are stored within adipocytes. Fat necrosis occurs enzymatically and non-enzymatically. In acute pancreatitis, saponification of peripancreatic fat occurs. During traumatic events, such as physical injury in breast tissue, non-enzymatic fat necrosis takes place. Option D: Protein breakdown into dipeptides and amino acids is the function of trypsin, not lipase. Lipoprotein lipase is found on the vascular endothelial surface and is responsible for degrading triglycerides that circulating from chylomicrons and VLDLs. Pancreatic lipase is found within the small intestine and is responsible for degrading dietary triglycerides.

For Rico who has chronic pancreatitis, which nursing intervention would be most helpful? A. Allowing liberalized fluid intake B. Counseling to stop alcohol consumption C. Encouraging daily exercise D. Modifying dietary protein

Correct Answer: B. Counseling to stop alcohol consumption. Chronic pancreatitis typically results from repeated episodes of acute pancreatitis. More than half of chronic pancreatitis cases are associated with alcoholism. Counseling to stop alcohol consumption would be the most helpful for the client. Explore the availability of treatment programs and rehabilitation of chemical dependency if indicated. Option A: Resume oral intake with clear liquids and advance diet slowly to provide a high-protein, high-carbohydrate diet, when indicated. Oral feedings given too early in the course of illness may exacerbate symptoms. Loss of pancreatic function and reduced insulin production may require the initiation of a diabetic diet. Option C: Daily exercise would be helpful but not the most beneficial intervention. Review the importance of initially continuing a bland, low-fat diet with frequent small feedings and restricted caffeine, with a gradual resumption of a normal diet within individual tolerance. Option D: Dietary protein modification is not necessary for chronic pancreatitis. Maintain NPO status and gastric suctioning in the acute phase. Prevents stimulation and release of pancreatic enzymes (secretin), released when chyme and HCl enter the duodenum.

When evaluating a male client for complications of acute pancreatitis, the nurse would observe for: A. Increased intracranial pressure B. Decreased urine output C. Bradycardia D. Hypertension

Correct Answer: B. Decreased urine output Acute pancreatitis can cause decreased urine output, which results from the renal failure that sometimes accompanies this condition. AKI develops late in the course of acute pancreatitis, usually after failure of other organs. Remarkably, the kidney was the first organ to fail in only 8.9% of patients with AKI, and only a minority of patients develop isolated AKI Option A: Intracranial pressure neither increases nor decreases in a client with pancreatitis. The causes of increased intracranial pressure (ICP) can be divided based on the intracerebral components causing elevated pressures. Generalized swelling of the brain or cerebral edema from a variety of causes such as trauma, ischemia, hyperammonemia, uremic encephalopathy, and hyponatremia. Option C: Tachycardia, not bradycardia, usually is associated with pulmonary or hypovolemic complications of pancreatitis. Tachycardia and mild hypotension may result from hypovolemia from sequestration of fluid in the pancreatic bed. About 60% of patients develop low-grade pyrexia from peripancreatic inflammation without evident infection. Option D: Hypotension can be caused by a hypovolemic complication, but hypertension usually isn't related to acute pancreatitis. Release into the systemic circulation of activated enzymes and proteases may cause endothelial damage leading to extravasation of fluids from the vascular space, hypovolemia, hypotension, increased abdominal pressure, intense kidney vasoconstriction, hypercoagulability, and fibrin deposition in the glomeruli.

You're caring for Lewis, a 67 y.o. patient with liver cirrhosis who developed ascites and requires paracentesis. Relief of which symptom indicates that the paracentesis was effective? A. Pruritus B. Dyspnea C. Jaundice D. Peripheral Neuropathy

Correct Answer: B. Dyspnea Ascites put pressure on the diaphragm. Paracentesis is done to remove fluid and reduce pressure on the diaphragm. The goal is to improve the patient's breathing. The others are signs of cirrhosis that aren't relieved by paracentesis. Dyspnea from tense ascites might only be relieved with large-volume paracentesis. Care should also be taken with this procedure because patients with cirrhosis who have unrecognized cardiomyopathy can develop pulmonary edema following large-volume paracentesis. Option A: These medical treatments range from bile salts, rifampicin, and opioid receptor antagonists to antihistamines. Additionally, nonpharmacological management such as skin moisturizers, avoidance of skin irritants, and avoiding hot environments can also prove to be very beneficial in reducing pruritus. Option C: The underlying disorder and any problems it causes are treated as needed. If jaundice is due to acute viral hepatitis, it may disappear gradually, without treatment, as the condition of the liver improves. However, hepatitis may become chronic, even if jaundice disappears. Jaundice itself requires no treatment in adults Option D: Treatment of peripheral neuropathy has two goals: controlling the underlying disease process and treating troublesome symptoms. The former is usually achieved by eliminating offending agents, such as toxins or medications; correcting a nutritional deficiency; or treating the underlying disease.

A client is suspected of having hepatitis. Which diagnostic test result will assist in confirming this diagnosis? A. Elevated hemoglobin level B. Elevated serum bilirubin level C. Elevated blood urea nitrogen level D. Decreased erythrocyte sedimentation rate

Correct Answer: B. Elevated serum bilirubin level. Laboratory indicators of hepatitis include elevated liver enzyme levels, elevated serum bilirubin levels, elevated erythrocyte sedimentation rates, and leukopenia. Baseline evaluation in a patient suspected to have viral hepatitis can be started by checking a hepatic function panel. Patients who have a severe disease can have elevated total bilirubin levels. Typically, levels of alkaline phosphatase (ALP) remain in the reference range, but if it is elevated significantly, the clinician should consider biliary obstruction or liver abscess. Option A: A hemoglobin level is unrelated to this diagnosis. In advanced liver disease, prothrombin time (PT) and international normalized ratio (INR) may appear prolonged. Patients may also have leukopenia and thrombocytopenia. Patients who suffer from easy bruising, variceal bleed, or hemorrhoidal bleed due to advanced liver disease may have anemia with low hemoglobin and hematocrit levels. Option C: An elevated blood urea nitrogen level may indicate renal dysfunction. Blood urea nitrogen (BUN) and serum creatinine levels are also necessary for patients suspected to have advanced liver disease to look for renal impairment. Patients who present with altered mental status should have serum ammonia levels checked and are usually elevated in the presence of hepatic encephalopathy. Option D: Elevated erythrocyte sedimentation rate is a laboratory indicator of hepatitis. The increase in the ESR in type A hepatitis could be explained by changes in the serum protein levels in the course of acute viral hepatitis and/or by the different inflammatory activity of the underlying disease.

What laboratory finding is the primary diagnostic indicator for pancreatitis? A. Elevated blood urea nitrogen (BUN) B. Elevated serum lipase C. Elevated aspartate aminotransferase (AST) D. Increased lactate dehydrogenase (LD)

Correct Answer: B. Elevated serum lipase Elevation of serum lipase is the most reliable indicator of pancreatitis because this enzyme is produced solely by the pancreas. Serum lipase typically increases 3-6 hours after the onset of acute pancreatitis and usually peaks at 24 hours. Unlike amylase, there is significant reabsorption of lipase in the renal tubules so the serum concentrations remain elevated for 8-14 days. Option A: A client's BUN is typically elevated in relation to renal dysfunction. A BUN test is done to see how well the kidneys are working. If the kidneys are not able to remove urea from the blood normally, the BUN level rises. Heart failure, dehydration, or a diet high in protein can also make the BUN level higher. Liver disease or damage can lower the BUN level. Option C: A client's AST is typically elevated in relation to liver dysfunction. The elevated AST-to-ALT ratio in alcoholic liver disease results in part from the depletion of vitamin B6 (pyridoxine) in chronic alcoholics. ALT and AST both use pyridoxine as a coenzyme, but the synthesis of ALT is more strongly inhibited by pyridoxine deficiency than is the synthesis of AST. Option D: A client's LD is typically elevated in relation to damaged cardiac muscle. Usually, LDH isoenzyme levels increase 24-72 hours following myocardial infarction and reach a peak concentration in 3-4 days. Glycogen phosphorylase BB is released into circulation 2-4 h after onset of cardiac ischemia and returns to baseline levels 1-2 days after acute myocardial infarction, making it an early marker.

Ralph has a history of alcohol abuse and has acute pancreatitis. Which lab value is most likely to be elevated? A. Calcium B. Glucose C. Magnesium D. Potassium

Correct Answer: B. Glucose Glucose level increases and diabetes mellitus may result d/t the pancreatic damage to the islets of Langerhans. Acute pancreatitis is associated with damage to both the endocrine and exocrine pancreas. Glucose intolerance seen with this disease appears to be the result of hyperglucagonemia and relative hypoinsulinemia. Option A: Initial evaluation of suspected acute pancreatitis involves laboratory abnormalities suggesting biliary cholestasis, hypercalcemia or severe hyperlipidemia will help in determining the etiology of pancreatitis. An abdominal ultrasound is recommended in all the patients to assess for choledocholithiasis and bile duct dilatation. Option C: The diagnosis of acute pancreatitis has been defined by the Revised Atlanta Classification and requires at least 2 of 3 criteria be met: 1) a lipase or amylase level that is three times the upper limit of normal 2) abdominal pain that is consistent with pancreatitis 3) abdominal imaging consistent with acute pancreatitis. Option D: A thorough history regarding alcohol use and medications should be gathered, keeping in mind that over five years of heavy alcohol use is often needed to induce alcohol-related pancreatitis. Smoking history is also important as a risk factor for acute pancreatitis.

Claire, a 33 y.o. is on your floor with a possible bowel obstruction. Which intervention is a priority for her? A. Obtain daily weights. B. Measure abdominal girth. C. Keep strict intake and output. D. Encourage her to increase fluids.

Correct Answer: B. Measure abdominal girth. Measuring abdominal girth provides quantitative information about increases or decreases in the amount of distention. Abdominal girths should be measured daily. Use the same measuring tape each time. Place the patient in the same position each time. Ensure that the tape measure is placed in the same position each time. This can be done by drawing small tick marks on the patient's abdomen to indicate the position of the tape. Measure the patient at the same time each day. Option A: Weigh daily; provides information about dietary needs and effectiveness of therapy. Avoid or limit foods that might cause or exacerbate abdominal cramping, flatulence (milk products, foods high in fiber or fat, alcohol, caffeinated beverages, chocolate, peppermint, tomatoes, orange juice). Option C: Monitor I&O closely. Fluid and electrolyte losses must be replaced. Record intake and changes in symptomatology. Useful in identifying specific deficiencies and determining GI response to foods. Monitor I&O. Note number, character, and amount of stools; estimate insensible fluid losses (diaphoresis). Measure urine specific gravity; observe for oliguria. Option D: Administer parenteral fluids, blood transfusions as indicated. Maintenance of bowel rest requires alternative fluid replacement to correct losses and anemia. Fluids containing sodium may be restricted in presence of regional enteritis.

When planning home care for a client with hepatitis A, which preventive measure should be emphasized to protect the client's family? A. Keeping the client in complete isolation B. Using good sanitation with dishes and shared bathrooms C. Avoiding contact with blood-soiled clothing or dressing D. Forbidding the sharing of needles or syringes

Correct Answer: B. Using good sanitation with dishes and shared bathrooms. Hepatitis A is transmitted through the fecal-oral route or from contaminated water or food. Measures to protect the family include good handwashing, personal hygiene and sanitation, and the use of standard precautions. According to the WHO, the most effective way to prevent HAV infection is to improve sanitation, food safety, and immunization practices. Option A: Complete isolation is not required. No specific treatment is needed for most patients with acute, uncomplicated HAV infection beyond supportive care. Complete recovery from symptoms may take several weeks to months. Option C: HAV is most commonly transmitted through the oral-fecal route via exposure to contaminated food, water, or close physical contact with an infectious person. According to the World Health Organization (WHO), infection rates in developed countries are low. Option D: Avoiding the sharing of needles or syringes are precautions needed to prevent transmission of hepatitis B. Globally, the rates of HAV have decreased due to improvements in public healthcare policies, sanitation, and education, but infection rates of other hepatitis viruses appear to be increasing.

Nurse Berlinda is assigned to a 41-year-old client who has a diagnosis of chronic pancreatitis. The nurse reviews the laboratory result, anticipating a laboratory report that indicates a serum amylase level of: A. 45 units/L B. 100 units/L C. 300 units/L D. 500 units/L

Correct Answer: C. 300 units/L The normal serum amylase level is 25 to 151 units/L. With chronic cases of pancreatitis, the rise in serum amylase levels usually does not exceed three times the normal value. In acute pancreatitis, the value may exceed five times the normal value. Basic lab studies for chronic pancreatitis can include a CBC, BMP, LFTs, lipase, amylase, lipid panel, and a fecal-elastase-1 value. Lipase and amylase levels can be elevated, but they are usually normal secondary to significant pancreatic scarring and fibrosis. Of note, amylase and lipase values should not be considered diagnostic or prognostic. Option A: 45 units/L is within normal limits. Serum amylase and lipase levels may be slightly elevated in chronic pancreatitis; high levels are found only during acute attacks of pancreatitis. In the later stages of chronic pancreatitis, atrophy of the pancreatic parenchyma can result in normal serum enzyme levels because of significant fibrosis of the pancreas, resulting in decreased concentrations of these enzymes within the pancreas. Option B: 100 units/L is within normal limits. When pancreatic tissue damage (eg. pancreatitis) or pancreatic duct is blocked, serum amylase levels increased. In acute pancreatitis, lipase levels are often very high; 10.5 times the normal level can be increased Option D: 500 units/L is an extremely elevated level seen in acute pancreatitis. In acute pancreatitis, blood amylase increased. Sometimes up to 4-6 times the highest normal level rises.

A client with advanced cirrhosis has been diagnosed with hepatic encephalopathy. The nurse expects to assess for: A. Malaise B. Stomatitis C. Hand tremors D. Weight loss

Correct Answer: C. Hand tremors Hepatic encephalopathy results from the accumulation of neurotoxins in the blood, therefore the nurse wants to assess for signs of neurological involvement. Flapping of the hands (asterixis), changes in mentation, agitation, and confusion are common. During the intermediate stages of HE, a characteristic jerking movement of the limbs is often observed (e.g., asterixis) when the patient attempts to hold arms outstretched with hands bent upward at the wrist. Option A: Malaise is not related to neurological involvement. Other physical signs may include hyperreflexia, a positive Babinski's sign, or Parkinsonian symptoms (e.g., rigidity or tremors). Symptoms typically include confusion, personality changes, disorientation, and a depressed level of consciousness. The earliest stage is often characterized by an inverted sleep-wake pattern wherein patients are found to be sleeping during the day and awake throughout the night. Option B: Stomatitis is not related to neurological involvement. Throughout the intermediate stages, patients tend to experience worsening levels of confusion, lethargy, and personality changes. In the advanced stages, hepatic encephalopathy may eventually lead to coma (e.g., hepatic coma or coma hepaticum) and ultimately to death. Option D: These clients typically have ascites and edema so experience weight gain. In order to make a diagnosis of HE, there must be confirmed the presence of liver disease (e.g., abnormal liver function tests, ultrasound or liver biopsy demonstrating liver disease) or a portosystemic shunt, and exclusion of other potential etiologies (e.g., intracranial lesions, masses, hemorrhage or stroke; seizure activity; post-seizure encephalopathy; intracranial infections; or toxic encephalopathy from other causes).

Marie, a 51-year-old woman, is diagnosed with cholecystitis. Which diet, when selected by the client, indicates that the nurse's teaching has been successful? A. 4-6 small meals of low-carbohydrate foods daily B. High-fat, high-carbohydrate meals C. Low-fat, high-carbohydrate meals D. High-fat, low protein meals

Correct Answer: C. Low-fat, high-carbohydrate meals Option C: For the client with cholecystitis, fat intake should be reduced. The calories from fat should be substituted with carbohydrates. Option A: Reducing carbohydrate intake would be contraindicated. Options B & D: Any diet high in fat may lead to another attack of cholecystitis.

Pierre, who is diagnosed with acute pancreatitis, is under the care of Nurse Bryan. Which intervention should the nurse include in the care plan for the client? A. Administration of vasopressin and insertion of a balloon tamponade B. Preparation for a paracentesis and administration of diuretics C. Maintenance of nothing-by-mouth status and insertion of nasogastric (NG) tube with low intermittent suction D. Dietary plan of a low-fat diet and increased fluid intake to 2,000 ml/day

Correct Answer: C. Maintenance of nothing-by-mouth status and insertion of nasogastric (NG) tube with low intermittent suction With acute pancreatitis, the client is kept on nothing-by-mouth status to inhibit pancreatic stimulation and secretion of pancreatic enzymes. NG intubation with low intermittent suction is used to relieve nausea and vomiting, decrease painful abdominal distention, and remove hydrochloric acid. Prolonged bowel rest by nothing per os (NPO) to minimize pancreatic secretion was an important part of the therapy for any patient with acute pancreatitis. Option A: Vasopressin would be appropriate for a client diagnosed with bleeding esophageal varices. The most common cause of late death in acute necrotizing pancreatitis is represented by organ failure through infected pancreatic necrosis (IPN). Therefore there might be a theoretical benefit from antibiotic prophylaxis. Option B: Paracentesis and diuretics would be appropriate for a client diagnosed with portal hypertension and ascites. Fluid therapy in acute pancreatitis can be seen as double edge sword with risk of necrosis through tissue hypoperfusion by using low fluid quantities and liquid sequestration and increased morbidity with too high volumes Option D: A low-fat diet and increased fluid intake would further aggravate pancreatitis. The concept of nutritional support in AP has gradually moved towards enteral feeding, due to large evidence proving safety and efficiency. Timing and mode of nutritional support in acute pancreatitis should be based on risk prediction of severity.

The nurse is reviewing the physician's orders written for a male client admitted to the hospital with acute pancreatitis. Which physician order should the nurse question if noted on the client's chart? A. NPO status B. Nasogastric tube inserted C. Morphine sulfate for pain D. An anticholinergic medication

Correct Answer: C. Morphine sulfate for pain Meperidine (Demerol) rather than morphine sulfate is the medication of choice to treat pain because morphine sulfate can cause spasms in the sphincter of Oddi. Histological data show that treatment with morphine after induction of acute pancreatitis exacerbates the disease with increased pancreatic neutrophilic infiltration and necrosis in all three models of acute pancreatitis. Morphine also exacerbated acute pancreatitis-induced gut permeabilization and bacteremia. Option A: Historically, patients with acute pancreatitis would be kept without food by mouth (nil per os or NPO) until their physician team deemed them ready to eat again, usually based on blood tests or the reported level of pain. Option B: NGT is used if vomiting is a problem. The tube can be used for a few weeks. It can be used to remove fluid and air and give the pancreas more time to heal. It can also be used to put liquid food into the stomach as the client heals. Option D: Anticholinergics are used to a variable extent in the treatment of many gastrointestinal conditions, including acute or relapsing chronic pancreatitis, acid-peptic disorders of the upper gastrointestinal tract, chronic inflammations of the intestines, and so-called functional gastrointestinal disorders caused by excessive or abnormal motility.

Brenda, a 36 y.o. patient is on your floor with acute pancreatitis. Treatment for her includes: A. Continuous peritoneal lavage. B. Regular diet with increased fat. C. Nutritional support with TPN. D. Insertion of a T tube to drain the pancreas.

Correct Answer: C. Nutritional support with TPN. With acute pancreatitis, you need to rest the GI tract by TPN as nutritional support. In cases of severe pancreatitis or where peroral intake is not tolerated, nasojejunal feeding is superior to parenteral nutrition as it helps to minimize bacterial translocation by maintaining the intestinal barrier. Option A: The foundation of management for acute pancreatitis remains early aggressive fluid resuscitation. Lactated Ringer's solution is the recommended fluid with an initial bolus of 15 to 20 mL/kg and following rates of 3 mL/kg per hour (usually approximately 250 to 500 mL per hour) for the first 24 hours if no other contraindications are present. Option B: Common practice is to keep nothing by mouth until abdominal pain, nausea, vomiting, appetite, and ileus improve. Early feeding in mild pancreatitis is safe and does not exacerbate symptoms. Soft, low residue, low-fat diet is recommended for initial feeding and advanced to regular consistency as tolerated. Option D: Further management depends upon the etiology of pancreatitis. In gallstone pancreatitis, early cholecystectomy is strongly recommended. Early ERCP (within 24 hours of presentation) is of benefit in cases of concurrent cholangitis and obvious biliary obstruction.

The nurse is caring for a male client with cirrhosis. Which assessment findings indicate that the client has deficient vitamin K absorption caused by this hepatic disease? A. Dyspnea and fatigue B. Ascites and orthopnea C. Purpura and petechiae D. Gynecomastia and testicular atrophy

Correct Answer: C. Purpura and petechiae A hepatic disorder, such as cirrhosis, may disrupt the liver's normal use of vitamin K to produce prothrombin (a clotting factor). Consequently, the nurse should monitor the client for signs of bleeding, including purpura and petechiae. Petechiae and purpura result from a wide variety of underlying disorders and may occur at any age. Petechiae are small (1-3 mm), red, non-blanching macular lesions caused by intradermal capillary bleeding. Purpura are larger, typically raised lesions resulting from bleeding within the skin Option A: Dyspnea and fatigue suggest anemia. When patients with severe anemia develop a high output state, cardiac failure can ensue causing them to have shortness of breath from cardiac causes as well. Diseases in other organs, such as the kidneys and the liver, may cause dyspnea by a combination of the interactions discussed. Option B: Ascites and orthopnea are unrelated to vitamin K absorption. Patients with malignant ascites can have symptoms related to malignancy, which may include weight loss. On the other hand, patients with ascites due to heart failure may report dyspnea, orthopnea, and peripheral edema, and those with chylous ascites report diarrhea, steatorrhea, malnutrition, edema, nausea, enlarged lymph nodes, early satiety, fevers, and night sweats. Option D: Gynecomastia and testicular atrophy result from decreased estrogen metabolism by the diseased liver. Gynecomastia is most commonly caused by an imbalance between the hormones estrogen and testosterone. Estrogen controls female traits, including breast growth. Testosterone controls male traits, such as muscle mass and body hair.

Britney, a 20 y.o. student is admitted with acute pancreatitis. Which laboratory findings do you expect to be abnormal for this patient? A. Serum creatinine and BUN B. Alanine aminotransferase (ALT) and aspartate aminotransferase (AST) C. Serum amylase and lipase D. Cardiac enzymes

Correct Answer: C. Serum amylase and lipase Pancreatitis involves the activation of pancreatic enzymes, such as amylase and lipase. These levels are elevated in a patient with acute pancreatitis. The diagnosis of acute pancreatitis has been defined by the Revised Atlanta Classification and requires at least 2 of 3 criteria to be met: 1) a lipase or amylase level that is three times the upper limit of normal 2) abdominal pain that is consistent with pancreatitis 3) abdominal imaging consistent with acute pancreatitis. Option A: Early changes in BUN level may reflect several important physiologic processes in acute pancreatitis. In addition to intravascular volume depletion, a rise in BUN level may be secondary to impairment in renal function or potentially concurrent upper gastrointestinal hemorrhage. Renal failure is a relatively common form of organ dysfunction among patients with acute pancreatitis Option B: Jaundice with increased ALT suggests gallstone etiology requiring ERCP. ALT or AST levels more than three times the upper limit of normal indicates gallstones as the cause of acute pancreatitis. However, the absence of elevated transaminases does not rule out gallstones. ALT has high specificity, but low sensitivity for gallstone pancreatitis. Option D: Acute pancreatitis can be associated with electrical changes mimicking acute coronary syndrome with normal coronary arteries. The association of acute pancreatitis with ST-segment elevation and elevated cardiac enzymes has been reported in few observations. The pathophysiological mechanisms of this association remain poorly understood.

A female client with viral hepatitis A is being treated in an acute care facility. Because the client requires enteric precautions, the nurse should: A. Place the client in a private room. B. Wear a mask when handling the client's bedpan. C. Wash the hands after touching the client. D. Wear a gown when providing personal care for the client.

Correct Answer: C. Wash the hands after touching the client. To maintain enteric precautions, the nurse must wash the hands after touching the client or potentially contaminated articles and before caring for another client. Enteric precautions are taken to prevent infections that are transmitted primarily by direct or indirect contact with fecal material. They're indicated for patients with known or suspected infectious diarrhea or gastroenteritis. Option A: A private room is warranted only if the client has poor hygiene — for instance, if the client is unlikely to wash the hands after touching infective material or is likely to share contaminated articles with other clients. Option B: For enteric precautions, the nurse need not wear a mask. While taking care of the client, the nurse may wear gloves and a gown if she might have contact with body fluids (stool, urine, saliva). A sign on the door to the client's room reminds staff to wear a gown and gloves when inside the room. Staff will wash their hands before entering and leaving the client's room. Option D: For enteric precautions, the nurse must wear a gown only if soiling from fecal matter is likely. Wash hands before entering and before leaving the client's room. Make sure to use soap and water when leaving the room. Be sure other visitors do this too.

The nurse caring for a client with small bowel obstruction would plan to implement which nursing intervention first? A. Administering pain medication B. Obtaining a blood sample for laboratory studies C. Preparing to insert a nasogastric (NG) tube D. Administering I.V. fluids

Correct Answer: D. Administering I.V. fluids. I.V. infusions containing normal saline solution and potassium should be given first to maintain fluid and electrolyte balance. Maintenance of bowel rest requires alternative fluid replacement to correct losses and anemia. Fluids containing sodium may be restricted in presence of regional enteritis. Option A: Pain medication often is withheld until the obstruction is diagnosed because analgesics can decrease intestinal motility. Provide comfort measures (back rub, reposition) and diversional activities. Promotes relaxation, refocuses attention, and may enhance coping abilities. Option B: A blood sample is then obtained for laboratory studies to aid in the diagnosis of bowel obstruction and guide treatment. Blood studies usually include a complete blood count, serum electrolyte levels, and blood urea nitrogen level. Option C: For the client's comfort and to assist in bowel decompression, the nurse should prepare to insert an NG tube next. Resume or advance diet as indicated (clear liquids progressing to bland, low residue; then high-protein, high-calorie, caffeine-free, non-spicy, and low-fiber as indicated).

A female client with hepatitis C develops liver failure and GI hemorrhage. The blood products that would most likely bring about hemostasis in the client are: A. Whole blood and albumin. B. Platelets and packed red blood cells. C. Fresh frozen plasma and whole blood. D. Cryoprecipitate and fresh frozen plasma.

Correct Answer: D. Cryoprecipitate and fresh frozen plasma. The liver is vital in the synthesis of clotting factors, so when it's diseased or dysfunctional, as in hepatitis C, bleeding occurs. Treatment consists of administering blood products that aid clotting. These include fresh frozen plasma containing fibrinogen and cryoprecipitate, which have most of the clotting factors. Option A: Although administering whole blood, albumin, and packed cells will contribute to hemostasis, those products aren't specifically used to treat hemostasis. Whole blood is often divided into component parts for ease of storage and administration. These typically include Red Blood Cells (RBC), Platelets (thrombocytes), and Plasma. Option B: Platelets are helpful, but the best answer is cryoprecipitate and fresh frozen plasma. Platelets are typically given when patients have a low platelet count (thrombocytopenia) or have platelets that are dysfunctional, due to medications or other acquired or inherited lesions. Option C: The indications for whole blood and blood component transfusion consist of increasing hemoglobin and oxygenation of tissues, maintaining adequate blood volume to avoid ischemia and hypovolemic shock, and to reconstitute platelets, coagulation factors, and other plasma proteins to a functional status.

What assessment finding of a patient with acute pancreatitis would indicate a bluish discoloration around the umbilicus? A. Grey-Turner's sign B. Homan's sign C. Rovsing's sign D. Cullen's sign

Correct Answer: D. Cullen's sign Cullen's sign is associated with pancreatitis when a hemorrhage is suspected. Cullen's sign is described as superficial edema with bruising in the subcutaneous fatty tissue around the periumbilical region. It is also known as periumbilical ecchymosis. It is most often recognized as a result of hemorrhagic pancreatitis. The sign can take 2-3 days before appearance and may be used as a clinical sign to help the diagnosis of acute pancreatitis. Option A: Grey-Turner's sign is ecchymosis in the flank area suggesting retroperitoneal bleed. Grey Turner's sign is an uncommon subcutaneous manifestation of intra-abdominal pathology that manifests as ecchymosis or discoloration of the flanks. Classically it correlates with severe acute necrotizing pancreatitis, often in association with Cullen's sign (periumbilical ecchymosis). Option B: Homan's sign is called pain elicited by the dorsiflexion of the foot and suggests deep vein thrombosis. Homan's sign test also called dorsiflexion sign test is a physical examination procedure that is used to test for deep vein thrombosis (DVT). A positive Homan's sign in the presence of other clinical signs may be a quick indicator of DVT. Clinical evaluation alone cannot be relied on for patient management, but when carefully performed, it remains useful in determining the need for additional testing (like D-dimer test, ultrasonography, multidetector helical computed axial tomography (CT), and pulmonary angiography). Option C: Rovsing's sign is associated with appendicitis when pain is felt with pressure at McBurney's point. Rovsing's sign is a clinical finding that is indicative of acute appendicitis (the inflammation and possible infection of the appendix). A positive Rovsing's sign is characterized by right lower abdominal pain upon palpation of the left side of the lower abdomen

Which clinical manifestation would the nurse expect a client diagnosed with acute cholecystitis to exhibit? A. Jaundice, dark urine, and steatorrhea B. Acute right lower quadrant (RLQ) pain, diarrhea, and dehydration C. Ecchymosis petechiae, and coffee-ground emesis D. Nausea, vomiting, and anorexia

Correct Answer: D. Nausea, vomiting, and anorexia Acute cholecystitis is an acute inflammation of the gallbladder commonly manifested by the following: anorexia, nausea, and vomiting; biliary colic; tenderness and rigidity the right upper quadrant (RUQ) elicited on palpation (e.g., Murphy's sign); fever; fat intolerance; and signs and symptoms of jaundice. Option A: Jaundice, dark urine, and steatorrhea are clinical manifestations of the icteric phase of hepatitis. Patients in this phase present with dark-colored urine and pale-colored stool. Some patients develop jaundice and right upper quadrant pain with liver enlargement. Option B: Cases of chronic cholecystitis present with progressing right upper quadrant abdominal pain with bloating, food intolerances (especially greasy and spicy foods), increased gas, nausea, and vomiting. Pain in the mid back or shoulder may also occur. Option C: Ecchymosis, petechiae, and coffee-ground emesis are clinical manifestations of esophageal bleeding. The coffee-ground appearance indicates old bleeding. The clinical presentation can vary but should be well-characterized. Hematemesis is the overt bleeding with vomiting of fresh blood or clots. Melena refers to dark and tarry-appearing stools with a distinctive smell. The term "coffee-grounds" describes gastric aspirate or vomitus that contains dark specks of old blood.

Which phase of hepatitis would the nurse incur strict precautionary measures at? A. Icteric B. Non-icteric C. Post-icteric D. Pre-icteric

Correct Answer: D. Pre-icteric Pre-icteric is the infective phase and precautionary measures should be strictly enforced. However, most patients are not always diagnosed during this phase. Nonspecific symptoms occur; they include profound anorexia, malaise, nausea and vomiting, a newly developed distaste for cigarettes (in smokers), and often fever or right upper quadrant abdominal pain. Urticaria and arthralgias occasionally occur, especially in HBV infection. Option A: During the icteric phase, precautionary measures should already be in place. After 3 to 10 days, the urine darkens, followed by jaundice. Systemic symptoms often regress, and patients feel better despite worsening jaundice. The liver is usually enlarged and tender, but the edge of the liver remains soft and smooth. Mild splenomegaly occurs in 15 to 20% of patients. Jaundice usually peaks within 1 to 2 weeks. Option B: There is no non-icteric phase. Some manifestations of acute hepatitis are virus-specific, but in general, acute infection tends to develop in predictable phases. Acute viral hepatitis is a common, worldwide disease that has different causes; each type shares clinical, biochemical, and morphologic features. The term acute viral hepatitis often refers to infection of the liver by one of the hepatitis viruses. Option C: During the post-icteric phase, precautionary measures should already be in place. During this 2- to 4-week period, jaundice fades. Appetite usually returns after the first week of symptoms. Acute viral hepatitis usually resolves spontaneously 4 to 8 weeks after symptom onset.

A client diagnosed with chronic cirrhosis who has ascites and pitting peripheral edema also has hepatic encephalopathy. Which of the following nursing interventions are appropriate to prevent skin breakdown? Select all that apply. A. Range of motion every 4 hours B. Turn and reposition every 2 hours C. Abdominal and foot massages every 2 hours D. Alternating air pressure mattress E. Sit in chair for 30 minutes each shift

Correct Answers: B & D Edematous tissue must receive meticulous care to prevent tissue breakdown. An air pressure mattress, careful repositioning can prevent skin breakdown. Inspect pressure points and skin surfaces closely and routinely. Gently massage bony prominences or areas of continued stress. Use of emollient lotions and limiting use of soap for bathing may help. Option A: Range of motion exercises preserve joint function but do not prevent skin breakdown. Encourage and assist the patient with reposition on a regular schedule. Assist with active and passive ROM exercises as appropriate. Option B: Repositioning reduces pressure on edematous tissues to improve circulation. Exercises enhance circulation and improve and/or maintain joint mobility. Edematous tissues are more prone to breakdown and to the formation of decubitus. Ascites may stretch the skin to the point of tearing in severe cirrhosis. Option C: Abdominal or foot massage will not prevent skin breakdown but must be cleaned carefully to prevent breaks in skin integrity. Keep linens dry and free of wrinkles. Moisture aggravates pruritus and increases the risk of skin breakdown. Option D: Use an alternating pressure mattress, egg-crate mattress, waterbed, sheepskins, as indicated. Reduces dermal pressure, increases circulation, and diminishes the risk of tissue ischemia. Option E: The feet should be kept at the level of the heart or higher so Fowler's position should not be employed. Recommend elevating lower extremities. Enhances venous return and reduces edema formation in extremities.

The nurse is caring for a client with biliary obstruction. The nurse interprets that obstruction of which passage is related to the client's condition? Cystic duct Liver canaliculi Common bile duct Right hepatic duct

Cystic duct The gallbladder receives bile from the liver through the cystic duct. The liver collects bile in the canaliculi, from which bile flows into the right and left hepatic ducts and then into the common hepatic duct. From there, the bile can be transported for storage in the gallbladder through the cystic duct, or it can flow directly into the duodenum by way of the common bile duct.

The nurse is assessing a client 24 hours following a cholecystectomy. The nurse notes that the T-tube has drained 750 mL of green-brown drainage since the surgery. Which nursing intervention is most appropriate? Clamp the T-tube. Irrigate the T-tube. Document the findings. Notify the primary health care provider.

Document the findings. Following cholecystectomy, drainage from the T-tube is initially bloody and then turns a greenish-brown color. The drainage is measured as output. The amount of expected drainage will range from 500 to 1000 mL/day. The nurse would document the output.

The nurse is caring for a client with pancreatitis. Which finding should the nurse expect to note when reviewing the client's laboratory results? Elevated level of pepsin Decreased level of lactase Elevated level of amylase Decreased level of enterokinase

Elevated level of amylase The serum level of amylase, an enzyme produced by the pancreas, increases with pancreatitis. Amylase normally is responsible for carbohydrate digestion. Pepsin is produced by the stomach and is used in protein digestion. Lactase and enterokinase are enzymes produced by the small intestine; lactase splits lactose into galactose and fructose, and enterokinase activates trypsin.

The nurse is caring for a hospitalized client with pancreatitis. Which findings should the nurse look for and expect to note when reviewing the laboratory results? Select all that apply. Elevated lipase level Elevated lactase level Elevated trypsin level Elevated amylase level Elevated sucrase level

Elevated lipase level Elevated trypsin level Elevated amylase level Lipase, trypsin, and amylase are produced in the pancreas and aid in the digestion of fats, starches, and proteins, respectively. Lactase is produced in the small intestine and aids in splitting neutral fats into glycerol and fatty acids. Sucrase is produced in the small intestine and converts sucrose into glucose and fructose.

The nurse is reviewing the results of serum laboratory studies for a client admitted for suspected hepatitis. Which laboratory finding is most associated with hepatitis, requiring the nurse to contact the primary health care provider? Elevated serum bilirubin level Below normal hemoglobin concentration Elevated blood urea nitrogen (BUN) level Elevated erythrocyte sedimentation rate (ESR)

Elevated serum bilirubin level Laboratory indicators of hepatitis include elevated liver enzymes, serum bilirubin level, and ESR. However, ESR is a nonspecific test that indicates the presence of inflammation somewhere in the body. The hemoglobin concentration is unrelated to this diagnosis. An elevated BUN level may indicate renal dysfunction.

A client with cirrhosis complicated by ascites is admitted to the hospital. The client reports a 10-lb weight gain over the past 1½ weeks. The client has edema of the feet and ankles, and his abdomen is distended, taut, and shiny with striae. Which client problem is most appropriate at this time? Difficulty with sleeping Risk for skin breakdown Difficulty with breathing Excessive body fluid volume

Excessive body fluid volume The client with weight gain who also has cirrhosis complicated by ascites most often is retaining fluid. This is especially true when the client has not demonstrated an appreciable increase in food intake or when the weight gain is massive in relation to the time frame given. Therefore, excessive body fluid volume is the most appropriate problem. No data are given to support difficulty with breathing, although in some clients upward pressure on the diaphragm from ascites does impair respiration. Risk for skin breakdown assumes a lower priority because it is a risk rather than an actual problem. There are no data in the question that indicate that the client is having difficulty with sleep.

A client with chronic pancreatitis needs information on dietary modification to manage the health problem. Which item in the diet should the nurse teach the client to limit? Fat Protein Carbohydrate Water-soluble vitamins

Fat The client with chronic pancreatitis should limit fat in the diet and also take in small meals, which will reduce the amount of carbohydrates and protein that the client must digest at any one time. The client does not need to limit water-soluble vitamins in the diet.

A client is admitted to the hospital with acute viral hepatitis. Which sign or symptom should the nurse expect to note based on this diagnosis? Fatigue Pale urine Weight gain Spider angiomas

Fatigue Common manifestations of acute viral hepatitis include weight loss, dark urine, and fatigue. The client is anorexic, possibly from a toxin produced by the diseased liver, and finds food distasteful. The urine darkens because of excess bilirubin being excreted by the kidneys. Fatigue occurs during all phases of hepatitis. Spider angiomas-small, dilated blood vessels-are commonly seen in cirrhosis of the liver.

The nurse manager is providing an educational session to nursing staff members about the phases of viral hepatitis. The nurse manager tells the staff that which clinical manifestation(s) are primary characteristics of the preicteric phase? Pruritus Right upper quadrant pain Fatigue, anorexia, and nausea Jaundice, dark-colored urine, and clay-colored stools

Fatigue, anorexia, and nausea In the preicteric phase, the client has nonspecific complaints of fatigue, anorexia, nausea, cough, and joint pain. The remaining options are clinical manifestations that occur in the icteric phase. In the posticteric phase, jaundice decreases, the color of urine and stool returns to normal, and the client's appetite improves.

The nurse is assessing a client who is experiencing an acute episode of cholecystitis. Which of these clinical manifestations support this diagnosis? Select all that apply. Fever Positive Cullen's sign Complaints of indigestion Palpable mass in the left upper quadrant Pain in the upper right quadrant after a fatty meal Vague lower right quadrant abdominal discomfort

Fever Complaints of indigestion Pain in the upper right quadrant after a fatty meal During an acute episode of cholecystitis, the client may complain of severe right upper quadrant pain that radiates to the right scapula or shoulder or experience epigastric pain after a fatty or high-volume meal. Fever and signs of dehydration would also be expected, as well as complaints of indigestion, belching, flatulence, nausea, and vomiting. Options 4 and 6 are incorrect because they are inconsistent with the anatomical location of the gallbladder. Option 2 (Cullen's sign) is associated with pancreatitis.

The nurse is reviewing the primary health care provider's prescriptions written for a client admitted to the hospital with acute pancreatitis. Which prescription requires follow-up by the nurse? Full liquid diet Morphine sulfate for pain Nasogastric tube insertion An anticholinergic medication

Full liquid diet The client with acute pancreatitis is placed on NPO (nothing by mouth) status to decrease the activity of the pancreas, which occurs with oral intake. Pain management for acute pancreatitis typically begins with the administration of opioids by patient-controlled analgesia. Medications such as morphine or hydromorphone are typically used. Nasogastric tube insertion is done to provide suction of secretions and administer medications as necessary.

A client admitted to the hospital with a suspected diagnosis of acute pancreatitis is being assessed by the nurse. Which assessment findings would be consistent with acute pancreatitis? Select all that apply. Diarrhea Black, tarry stools Hyperactive bowel sounds Gray-blue color at the flank Abdominal guarding and tenderness Left upper quadrant pain with radiation to the back

Gray-blue color at the flank Abdominal guarding and tenderness Left upper quadrant pain with radiation to the back rayish-blue discoloration at the flank is known as Grey-Turner's sign and occurs as a result of pancreatic enzyme leakage to cutaneous tissue from the peritoneal cavity. The client may demonstrate abdominal guarding and may complain of tenderness with palpation. The pain associated with acute pancreatitis is often sudden in onset and is located in the epigastric region or left upper quadrant with radiation to the back. The other options are incorrect.

A client with viral hepatitis is having difficulty coping with the disorder. Which question by the nurse is the most appropriate in identifying the client's coping problem? "Do you have a fever?" "Are you losing weight?" "Have you enjoyed having visitors?" "Do you rest sometime during the day?"

Have you enjoyed having visitors?" Clients with hepatitis may experience anxiety because of an anticipated change in lifestyle or fear of prognosis. They also may have a disturbance in body image related to the stigma of having a communicable disease or a change in appearance because of jaundice. The correct option relates to the client's possible feelings of not wanting to be seen by others because of altered appearance. Remember that the client with hepatitis is jaundiced.

he nurse is caring for a client with acute pancreatitis and is monitoring the client for paralytic ileus. Which piece of assessment data should alert the nurse to this occurrence? Inability to pass flatus Loss of anal sphincter control Severe, constant pain with rapid onset Firm, nontender mass palpable at the lower right costal margin

Inability to pass flatus An inflammatory reaction such as acute pancreatitis can cause paralytic ileus, the most common form of nonmechanical obstruction. Inability to pass flatus is a clinical manifestation of paralytic ileus. Loss of sphincter control is not a sign of paralytic ileus. Pain is associated with paralytic ileus, but the pain usually manifests as a more constant generalized discomfort. Option 4 is the description of the physical finding of liver enlargement. The liver may be enlarged in cases of cirrhosis or hepatitis. Although this client may have an enlarged liver, an enlarged liver is not a sign of paralytic ileus or intestinal obstruction.

A client is diagnosed with viral hepatitis, complaining of "no appetite" and "losing my taste for food." What instruction should the nurse give the client to provide adequate nutrition? Select foods high in fat. Increase intake of fluids, including juices. Eat a good supper when anorexia is not as severe. Eat less often, preferably only 3 large meals daily.

Increase intake of fluids, including juices. Although no special diet is required to treat viral hepatitis, it is generally recommended that clients consume a low-fat diet, as fat may be tolerated poorly because of decreased bile production. Small, frequent meals are preferable and may even prevent nausea. Frequently, appetite is better in the morning, so it is easier to eat a good breakfast. An adequate fluid intake of 2500 to 3000 mL/day that includes nutritional juices is also important.

The nurse assists a primary health care provider in performing a liver biopsy. After the procedure, the nurse should place the client in which position? Prone Supine Left side Right side

Right side To splint and provide pressure at the puncture site, the client is kept on the right side for a minimum of 2 hours after a liver biopsy. Therefore, the remaining positions are incorrect.

The nurse is teaching the client with viral hepatitis about the stages of the disease. The nurse should explain to the client that the second stage of this disease is characterized by which specific assessment findings? Select all that apply. Jaundice Flu-like symptoms Clay-colored stools Elevated bilirubin levels Dark or tea-colored urine

Jaundice Clay-colored stools Elevated bilirubin levels Dark or tea-colored urine There are 3 stages associated with viral hepatitis. The first (preicteric) stage includes flu-like symptoms only. The second (icteric) stage includes the appearance of jaundice and associated symptoms such as elevated bilirubin levels, dark or tea-colored urine, and clay-colored stools. The third (posticteric) stage occurs when the jaundice decreases and the colors of the urine and stool return to normal.

The nurse is caring for a client with cirrhosis of the liver. To minimize the effects of the disorder, the nurse teaches the client about foods that are high in thiamine. The nurse determines that the client has the best understanding of the dietary measures to follow if the client states an intention to increase the intake of which food? Milk Chicken Broccoli Legumes

Legumes The client with cirrhosis needs to consume foods high in thiamine. Thiamine is present in a variety of foods of plant and animal origin. Legumes are especially rich in this vitamin. Other good food sources include nuts, whole-grain cereals, and pork. Milk contains vitamins A, D, and B2. Poultry contains niacin. Broccoli contains vitamins C, E, and K and folic acid.

A client with acute pancreatitis is experiencing severe pain from the disorder. The nurse determines that education about positioning to reduce pain was effective if the client avoids which action? Sitting up Lying flat Leaning forward Drawing the legs to the chest

Lying flat The pain of pancreatitis is aggravated by lying supine or walking. This is because the pancreas is located retroperitoneally, and the edema and inflammation will intensify the irritation of the posterior peritoneal wall with these positions or movements. Positions such as sitting up, leaning forward, and flexing the legs (especially the left leg) will alleviate some of the pain associated with pancreatitis. The fetal position (with the legs drawn up to the chest) may decrease the abdominal pain of pancreatitis.

The nurse is reviewing the prescription for a client admitted to the hospital with a diagnosis of acute pancreatitis. Which interventions would the nurse expect to be prescribed for the client? Select all that apply. Maintain NPO (nothing by mouth) status. Encourage coughing and deep breathing. Give small, frequent high-calorie feedings. Maintain the client in a supine and flat position. Give hydromorphone intravenously as prescribed for pain. Maintain intravenous fluids at 10 mL/hr to keep the vein open.

Maintain NPO (nothing by mouth) status. Encourage coughing and deep breathing. Give hydromorphone intravenously as prescribed for pain. The client with acute pancreatitis normally is placed on NPO status to rest the pancreas and suppress gastrointestinal secretions, so adequate intravenous hydration is necessary. Because abdominal pain is a prominent symptom of pancreatitis, pain medications such as morphine or hydromorphone are prescribed. Meperidine is avoided, as it may cause seizures. Some clients experience lessened pain by assuming positions that flex the trunk, with the knees drawn up to the chest. A side-lying position with the head elevated 45 degrees decreases tension on the abdomen and may help ease the pain. The client is susceptible to respiratory infections because the retroperitoneal fluid raises the diaphragm, which causes the client to take shallow, guarded abdominal breaths. Therefore, measures such as turning, coughing, and deep breathing are instituted.

A client has developed hepatitis A after eating contaminated oysters. The nurse assesses the client for which expected assessment finding? Malaise Dark stools Weight gain Left upper quadrant discomfort

Malaise Hepatitis causes gastrointestinal symptoms such as anorexia, nausea, right upper quadrant discomfort, and weight loss. Fatigue and malaise are common. Stools will be light- or clay-colored if conjugated bilirubin is unable to flow out of the liver because of inflammation or obstruction of the bile ducts.

The nurse is administering a cleansing enema to a client with a fecal impaction. Before administering the enema, the nurse should place the client in which position? Modified left lateral recumbent position Modified right lateral recumbent position On the left side of the body, with the head of the bed elevated 45 degrees On the right side of the body, with the head of the bed elevated 45 degrees

Modified left lateral recumbent position For administering an enema, the client is placed in a modified left lateral recumbent position so that the enema solution can flow by gravity in the natural direction of the colon. The head of the bed is not elevated in the Sims' position.

The nurse is creating a plan of care for a client with cirrhosis and ascites. Which nursing actions should be included in the care plan for this client? Select all that apply. Monitor daily weight. Measure abdominal girth. Monitor respiratory status. Place the client in a supine position. Assist the client with care as needed.

Monitor daily weight. Measure abdominal girth. Monitor respiratory status. Assist the client with care as needed. Ascites is a problem because as more fluid is retained, it pushes up on the diaphragm, thereby impairing the client's breathing patterns. The client should be placed in a semi-Fowler's position with the arms supported on a pillow to allow for free diaphragm movement. The correct options identify appropriate nursing interventions to be included in the plan of care for the client with ascites.

A client with viral hepatitis has no appetite, and food makes the client nauseated. Which nursing intervention is appropriate? Encourage foods that are high in protein. Monitor for fluid and electrolyte imbalance. Explain that high-fat diets usually are better tolerated. Explain that most daily calories need to be consumed in the evening hours.

Monitor for fluid and electrolyte imbalance. If nausea occurs and persists, the client will need to be assessed for fluid and electrolyte imbalance. It is important to explain to the client that most calories should be eaten in the morning hours because nausea is most common in the afternoon and evening. Clients should select a diet high in calories because energy is required for healing. Protein increases the workload on the liver. Changes in bilirubin interfere with fat absorption, so low-fat diets are better tolerated.

The nurse is caring for a client with acute pancreatitis. Which medications should the nurse expect to be prescribed for treatment of this problem? Select all that apply. Insulin Morphine Dicyclomine Pancrelipase Pantoprazole Acetazolamide

Morphine Dicyclomine Pantoprazole Acetazolamide Medications used to treat acute pancreatitis include pain medications such as morphine, antispasmodics such as dicyclomine, proton pump inhibitors such as pantoprazole, and acetazolamide to decrease the volume and bicarbonate concentration of pancreatic secretions. Insulin is used in chronic pancreatitis to treat diabetes mellitus or hyperglycemia if needed, and pancreatic enzyme products are used for replacement of pancreatic enzymes.

A client with cirrhosis has ascites and excess fluid volume. Which assessment findings does the nurse anticipate to note as a result of increased abdominal pressure? Select all that apply. Orthopnea and dyspnea Petechiae and ecchymosis Inguinal or umbilical hernia Poor body posture and balance Abdominal distention and tenderness

Orthopnea and dyspnea Petechiae and ecchymosis Inguinal or umbilical hernia Abdominal distention and tenderness Excess fluid volume, related to the accumulation of fluid in the peritoneal cavity and dependent areas of the body, can occur in the client with cirrhosis. Ascites can cause physical problems because of the overdistended abdomen and resultant pressure on internal organs and vessels. These problems include respiratory difficulty, petechiae and ecchymosis, development of hernias, and abdominal distention and tenderness. Poor body posture and balance are unrelated to increased abdominal pressure.

The nurse is reviewing the laboratory results for a client with cirrhosis and notes that the ammonia level is 85 mcg/dL (51 mcmol/L). Which dietary selection does the nurse suggest to the client? Roast pork Cheese omelet Pasta with sauce Tuna fish sandwich

Pasta with sauce Cirrhosis is a chronic, progressive disease of the liver characterized by diffuse degeneration and destruction of hepatocytes. The serum ammonia level assesses the ability of the liver to deaminate protein byproducts. Normal reference interval is 10 to 80 mcg/dL (6 to 47 mcmol/L). Most of the ammonia in the body is found in the gastrointestinal tract. Protein provided by the diet is transported to the liver by the portal vein. The liver breaks down protein, which results in the formation of ammonia. Foods high in protein should be avoided since the client's ammonia level is elevated above the normal range; therefore, pasta with sauce would be the best selection.

The nurse is assessing a client with liver disease for signs and symptoms of low albumin. Which sign or symptom should the nurse expect to note? Weight loss Peripheral edema Capillary refill of 5 seconds Bleeding from previous puncture sites

Peripheral edema Albumin is responsible for maintaining the osmolality of the blood. When the albumin level is low, osmotic pressure is decreased, which in turn can lead to peripheral edema. Weight loss is not a sign or symptom for hypoalbuminemia. Capillary refill of 5 seconds is a delayed filling time but is not associated with decreased albumin levels. Clotting factors produced by the liver (not albumin) are responsible for coagulation, and lack of clotting factors can result in bleeding from old puncture sites. The total protein level may decrease if the albumin level is low.

The nurse is monitoring a client with cirrhosis of the liver for signs of hepatic encephalopathy. Which assessment finding would the nurse note as an early sign of hepatic encephalopathy? Restlessness Presence of asterixis Complaints of fatigue Decreased serum ammonia levels

Presence of asterixis Asterixis is a flapping tremor of the hand that is an early sign of hepatic encephalopathy. The exact cause of this disorder is not known, but abnormal ammonia metabolism may be implicated. Increased serum ammonia levels are thought to interfere with normal cerebral metabolism. Tremors and drowsiness also would be noted.

A client with cirrhosis is beginning to show signs of hepatic encephalopathy. The nurse should plan a dietary consultation to limit the amount of which ingredient in the client's diet? Protein Calories Minerals Carbohydrates

Protein Ammonia is formed as a product of protein metabolism. Clients with hepatic encephalopathy have a high serum ammonia level, which is responsible for the symptoms of encephalopathy. Limiting protein intake will prevent further elevation in the serum ammonia level and prevent further deterioration of the client's mental status. It is not necessary to limit calories, minerals, or carbohydrates.

Lactulose is prescribed for a hospitalized client with a diagnosis of hepatic encephalopathy. Which assessment finding indicates that the client is responding to this medication therapy as anticipated? Vomiting occurs. The fecal pH is acidic. The client experiences diarrhea. The client is able to tolerate a full diet.

The fecal pH is acidic. Lactulose is an osmotic laxative used to decrease ammonia levels, which are elevated in hepatic encephalopathy. The desired effect is 2 or 3 soft stools per day with an acid fecal pH. Lactulose creates an acid environment in the bowel, resulting in a fall of the colon's pH from 7 to 5. This causes ammonia to leave the circulatory system and move into the colon for excretion. Diarrhea may indicate excessive administration of the medication. Vomiting and ability to tolerate a full diet do not determine that a desired effect has occurred.

A client is admitted to the hospital with a diagnosis of acute pancreatitis. Which would the nurse expect the client to report about the pain? Eating helps to decrease the pain. The pain usually increases after vomiting. The pain is mostly around the umbilicus and comes and goes. The pain increases when the client sits up and bends forward.

The pain usually increases after vomiting. Pain with acute pancreatitis usually increases after vomiting because of an increase in intraductal pressure caused by retching, which leads to further obstruction of the outflow of pancreatic secretions. The pain is a steady and intense epigastric pain that radiates to the client's back and flank. The pain may lessen when the client sits up or bends forward. Eating exacerbates the pain by stimulating the secretion of enzymes.

The nurse is obtaining a health history for a client with chronic pancreatitis. The health history is most likely to include which as a most common causative factor in this client's disorder? Weight gain Use of alcohol Exposure to occupational chemicals Abdominal pain relieved with food or antacids

Use of alcohol Chronic pancreatitis occurs most often in alcoholics. Abstinence from alcohol is important to prevent the client from developing chronic pancreatitis. Clients usually experience malabsorption with weight loss. Chemical exposure is associated with cancer of the pancreas. Pain will not be relieved with food or antacids.


संबंधित स्टडी सेट्स

05.04 Confidence Intervals for Proportions

View Set

Chapter 9 Test Questions AP Biology

View Set

Orientation: canvas student orientation quiz

View Set

Networking + 6 edition Chapter 4

View Set

AICPA Code of Professional Conduct: Conceptual Framework Part 1: Members in Public Practice (1B1 Video 2)

View Set